Integrating for logistic growth model

Click For Summary
The discussion focuses on solving a differential equation for the logistic growth model, specifically M'(t) = M(S-M) + I, where M represents biomass growth and I indicates immigration. The user is struggling with integrating the equation, particularly with the term involving immigration, and is unsure how to apply partial fractions to derive the general logistic equation. They express confusion over a specific step in the partial fraction decomposition, questioning the assumptions made about the constants A and B. A solution is provided, clarifying that by substituting specific values for M, one can determine A and B, leading to the correct partial fraction expansion. The conversation emphasizes the challenges of integrating logistic growth equations with additional terms like immigration.
Darkmisc
Messages
222
Reaction score
31

Homework Statement



Hi, can anyone help me solve a differential equation for the logistic growth model?

Homework Equations





It reads:

M'(t) = M(S-M) + I, where M(t) represents the growth of a biomass. "I" represents immigration (in a coral reef) and there is no breeding.




The Attempt at a Solution



I've treated it as a separable differential equation, but get the term

(integral of) 1/(M(S-M)+I) dM

A solution is possible using wolfram mathematica, but it doesn't isolate M.

I'd solve the equation using partial fractions, but the "I" term seems to make it impossible. Am I right in saying that?

Also, going back a step, can anyone explain how to use partial fractions to derive the general logistic equation. Wikipedia has a section on it: http://en.wikipedia.org/wiki/Partial_fraction

but there's a step I don't understand, namely: A=B, A=1/M, B=1/M. On what basis was this assumed?

In trying to solve it myself, I let A=0 and got B= 1/P, then let B=0 and got A=1/(M-P).

When these figures for A and B are substituted back into the original equation, I get 2/(P(M-P)) .

Can anyone explain what I'm doing wrong?


Thanks,

Darkmisc


 
Physics news on Phys.org
Darkmisc said:
Also, going back a step, can anyone explain how to use partial fractions to derive the general logistic equation. Wikipedia has a section on it: http://en.wikipedia.org/wiki/Partial_fraction

but there's a step I don't understand, namely: A=B, A=1/M, B=1/M. On what basis was this assumed?

In trying to solve it myself, I let A=0 and got B= 1/P, then let B=0 and got A=1/(M-P).

When these figures for A and B are substituted back into the original equation, I get 2/(P(M-P)) .

Can anyone explain what I'm doing wrong?

Say you are trying to set up this partial fraction expansion and figure out A and B:

\frac 1 {m(s-m)} = \frac A m+ \frac B {s-m}

Add the two fractions on the right to get:

\frac 1 {m(s-m)} = \frac {A(s-m)+Bm}{(m)(s-m)}

For these to be equal, the numerators must be equal:

1 = A(s-m)+Bm

If you put m = 0 you get A = 1/s and if you put m =s you get B = 1/s. So your partial fraction expansion becomes:

\frac 1 {m(s-m)} = \frac {\frac 1 s} m+ \frac {\frac 1 s} {s-m}
 
Question: A clock's minute hand has length 4 and its hour hand has length 3. What is the distance between the tips at the moment when it is increasing most rapidly?(Putnam Exam Question) Answer: Making assumption that both the hands moves at constant angular velocities, the answer is ## \sqrt{7} .## But don't you think this assumption is somewhat doubtful and wrong?

Similar threads

  • · Replies 3 ·
Replies
3
Views
2K
Replies
3
Views
2K
  • · Replies 1 ·
Replies
1
Views
1K
  • · Replies 10 ·
Replies
10
Views
6K
Replies
5
Views
2K
  • · Replies 8 ·
Replies
8
Views
2K
  • · Replies 7 ·
Replies
7
Views
2K
  • · Replies 4 ·
Replies
4
Views
2K
Replies
8
Views
1K
  • · Replies 7 ·
Replies
7
Views
3K